Complex variables: ML inequality

Click For Summary
The discussion centers on understanding a step in the proof of the ML inequality in complex analysis, which states that the integral of a continuous function over a contour is bounded by the product of the length of the contour and an upper bound of the function. The key point of confusion involves how the constant M, representing the upper bound of |f| on the contour, can be factored out of the integral. It is clarified that since M is an upper bound for |f(z)| on the contour, it can also be applied to |f(z(t))|, allowing the inequality to hold. This understanding resolves the initial confusion about the manipulation of the integral. The discussion concludes with a confirmation that the reasoning is sound and the participant feels satisfied with the explanation.
futurebird
Messages
270
Reaction score
0
I'm studying the proof of the ML inequlity from complex analysis. I don't know what they did in one step of the proof and I was wondering if anyone can explain the step to me.

First of all the theorem says:

Let f(z) be continuous on a contour C. Then

|\int_{c}f(z)dz| \leq ML

Where L is the length of C and M is an upper bound for |f| on C.

PROOF:
After a few steps we have:

|\int_{c}f(z)dz| \leq \int^{b}_{a} |f(z(t))||z'(t)|dt

And the text says " |f| is bounded on C, |f(z)| \leq M on C, where M is constant, then:

|\int_{c}f(z)dz| \leq M\int^{b}_{a} |z'(t)|dt

How did they get the M out of the integral?
 
Last edited:
Physics news on Phys.org
\int^{b}_{a} |f(z(t))||z'(t)|dt \leq \int_a^b M|z'(t)|dt = M \int_a^b |z'(t)|dt
 
So it's okay that M is an upper bound for |f(z)|, that's the same thing as an upper bound for |f(z(t))|? Okay thanks, I think I get it now.
 
Question: A clock's minute hand has length 4 and its hour hand has length 3. What is the distance between the tips at the moment when it is increasing most rapidly?(Putnam Exam Question) Answer: Making assumption that both the hands moves at constant angular velocities, the answer is ## \sqrt{7} .## But don't you think this assumption is somewhat doubtful and wrong?

Similar threads

Replies
1
Views
1K
Replies
8
Views
2K
  • · Replies 8 ·
Replies
8
Views
2K
Replies
6
Views
2K
  • · Replies 1 ·
Replies
1
Views
1K
  • · Replies 2 ·
Replies
2
Views
1K
  • · Replies 1 ·
Replies
1
Views
2K
  • · Replies 14 ·
Replies
14
Views
3K
  • · Replies 1 ·
Replies
1
Views
1K
Replies
9
Views
2K